If Horace can paint n houses in h hours. Which represents his average rate in houses per hour?

Answers

Answer 1

Answer:

n/h

Step-by-step explanation:

Average rate pf houses per hour = total number of houses painted / total hours

= n/h

Let's use an example - if i paint 10 houses in 2 hours.. how many houses would i paint in an hour

10/2 = 5 houses

Answer 2

Answer:

It's D, n/h

Step-by-step explanation:


Related Questions

Please Help Me, I will give the person who answers the best brainliest.

What is the equation of a line that contains the points (5, 0) and (5, −2)?

x = 5
x = 0
y = 0
y = 5

Answers

The answer has to be number 2 because I took the test and I got it right

math work pls help :)​

Answers

Answer:

e

Step-by-step explanation:

As the figure shows, the triangle are right triangle s. So the choice is e

p(x)=x^4 -2x^3 -11x +12x+36
The process of solving for all zeros (by hand) of the polynomial (work page)
The list of all solutions
The domain and range of the function
The end behavior of the function
The intervals for which the function is increasing and decreasing
Local Maximum(s)
Local Minimum(s)

Answers

x=−2x=-2 (Multiplicity of 22)

x=4x=4 (Multiplicity of 11)

About 35% of all U.S. households have a cellular phone. If you called 7 households to inquire if they have a cell phone.

To calculate this distribution, what would you set p equal to? p =
What would q be equal to? q =
What would n be equal to? n =​

Answers

Answer:

p = 0.35

q = 0.65

n = 7

Step-by-step explanation:

To calculate the distribution, we have the following values;

p = 35% = 35/100 = 0.35

q = 1-p = 1-0.35 = 0.65

n = 7

Answer:

p = 0.35

q = 0.65

n = 7

Step-by-step explanation:

Please help this is for a big grade

Answers

Answer:

x = 12

Side length = [tex]2\sqrt{3}[/tex] in

Step-by-step explanation:

Area of the given hexagon = 6 × (Area of the triangular section)

Area of the triangular section = [tex]\frac{1}{2}(\text{Base})(\text{Height})[/tex]

                                                 = [tex]\frac{1}{2}(2)(\sqrt{3})^{\frac{x}{12}}(\sqrt{3})^{\frac{x}{6}}[/tex]

                                                 = [tex](\sqrt{3})^{\frac{x}{12}}[(\sqrt{3})^2]^{\frac{x}{12}}[/tex]

                                                 = [tex](\sqrt{3})^{\frac{x}{12}}(3)^{\frac{x}{12}}[/tex]

                                                 = [tex](3\sqrt{3})^{\frac{x}{12}}[/tex]

Now area of the given hexagon = [tex]6(3\sqrt{3})^{\frac{x}{12}}[/tex]

Since, area of the hexagon is = [tex]18\sqrt{3}[/tex] in²

[tex]6(3\sqrt{3})^{\frac{x}{12}}=18\sqrt{3}[/tex]

[tex](3\sqrt{3})^{\frac{x}{12}}=(3\sqrt{3})^1[/tex]

[tex]\frac{x}{12}=1[/tex]

x = 12

Therefore, side length = [tex]2(\sqrt{3})^{\frac{x}{12}}[/tex]

                                      = [tex](2\sqrt{3})^{\frac{12}{12}}[/tex]

                                      = [tex]2\sqrt{3}[/tex] in.

find the radius of the circle. please.

Answers

Answer:

12in

Step-by-step explanation:

2r = 24

r = 12

Answer:

12 in.

Step-by-step explanation:

The radius is half of the diameter.

r = d/2

r = 24/2

r = 12

IM RELLY DONE I DONT UNDERSTAND THIS
And answer is not (-5.-2) e.t.c
PLS HELP​

Answers

Answer: -5, 1: -5, 2: -8, 1

Step-by-step explanation: Just flip it on the other side

Answer:

(2,5)

(1,5)

(1,8)

Step-by-step explanation:

The reflection line y = x does the following to the coordinates:

(x , y) --> (y , x)

The coordinates swap places.

(5,2)  --> (2,5)

(5,1)   --> (1,5)

(8,1)   --> (1,8)



1.Find the number such that twice its square root is 14

2.Find the number such that the square root of four more than five times the number is 8​

Answers

Answer:

rryvggugfèieuddbdbxkzbsvsjskbshsudisnssvehsij234

Answer:

1 ....ans is 49

as its root is 7 and double of 7 id 14

2 ....ans is

sq of 64 is 8

number is x

now

5x + 4 = 64

5x=60

x=12

so number is 12.


5) How many thousands are there in 2 158 621?​

Answers

Answer:

2,158,621 = 2 (i think)

2




Blah blah blah blah

Use the formula v= u + at to find the final velocity (in m/s), when
• the initial velocity is 3 m/s
• the acceleration is 2.5 m/s?
• the time is 12 seconds

Answers

v = u + at
v = 3 + (2.5x12)
v = 33m/s

A pool has a circular base with a diameter of 48 feet as shown in the diagram. What is the circumference of the base of the pool in feet?

Answers

Answer:

150.72

Step-by-step explanation:

C= pi*d

C= 3.14*48

C=150.72ft

if p ≥ q, then -p ≤ -q

Answers

Answer:

Yes, that's correct.

Step-by-step explanation:

What is the solution

-3x + y = 2

y = x + 4

Answers

Answer:

to find the solution, we substitute y = x + 4 into the other equation.

-3x + (x + 4) = 2

-3x + x + 4 = 2

-2x = 2 - 4

-2x = -2

x = 1

to find y, we substitute x to the x variable in the y equation.

y = 1 + 4

y = 5

therefore, when combined, it is (1,5).

giving brainlest to whoever can solve ^^

Answers

Answer:

c

Step-by-step explanation:

-8 + 12 =4

What is the distance from Point A to Point B?
A(-4,2)
B(6,-3)

Answers

Answer:

The distance is (10,5). Hope I helped! Have a great day :)

Step-by-step explanation:


Select the correct answer.

Which of the following statements is correct about quadratic number patterns?

A. The second difference is constant.

B. The third difference is greater than zero.

c. The first difference is constant.

D. The difference between terms is always positive.

Answers

Answer:

D.. I think..hope it helps

Hi! When I type into the calculator and multiply 0.12 and 100 it comes out as 13.2, why is that?

Answers

Answer:

because it is a decimal so it multiplies 100 by the decimal which is a smaller number, almost like how when you divide it becomes a smaller number.

Step-by-step explanation:

hope this helps:)

i'm not sure why that is, but the only thing i can assume is that either your calculator is broken somehow or you are accidentally typing in the wrong numbers.

you might also have forgotten to clear your calculator before attempting to put in 0.12 x 100.

i recommend using an online calculator for the time being if your calculator is faulty/having issues.

y = -63 y = 8x + 6
Use Substitution to solve​

Answers

9514 1404 393

Answer:

  (x, y) = (-8.625, -63)

Step-by-step explanation:

Substitute for y.

  -63 = 8x +6

  -69 = 8x . . . . . subtract 6

  -69/8 = x = -8.625

The solution is (x, y) = (-8.625, -63).

9x²+18x-7 (Factor COMPLETELY)

Answers

Answer:

(3x - 1) • (3x + 7)

Step-by-step explanation:

Also if you ever need help with algebra, tiger-algebra.com is a good way to solve it! It gives you the steps and everything too.

Hope this helps!

Step-by-step explanation:

[tex]\tt{}( {9x}^{2} - 18x - 7)[/tex]

[tex]\tt{} {9x}^{2} + 21 - 3x - 7[/tex]

[tex]\tt{}3x \times (3x + 7) -( 3x + 7)[/tex]

[tex]\tt{}(3x + 7) \times( 3x - 1)[/tex]

If f(x) = 11 - 6(x - 8), what is f(-3)?

Answers

Answer:

We conclude that:

[tex]f\left(-3\right)=77[/tex]

Step-by-step explanation:

Given the function

[tex]f(x) = 11 - 6(x - 8)[/tex]

Important Tip:

In order to determine f(-3), all we need is to substitute x = -3 in the function equation

substitute x = -3 in the equation

[tex]f(x) = 11 - 6(x - 8)[/tex]

[tex]f(-3) = 11 - 6(-3 - 8)[/tex]

[tex]f\left(-3\right)\:=\:11-\left(-66\right)[/tex]

[tex]f\left(-3\right)\:=\:11+66[/tex]               ∵ Apply rule  -(-a) = a

[tex]f\left(-3\right)=77[/tex]

Therefore, we conclude that:

[tex]f\left(-3\right)=77[/tex]

5x + 3y = 6
-3x - 3y = 6

Answers

Answer:

(x,y)=(6,-8)

x=6, y=-8

Step-by-step explanation:

There are many was to answer this question, some are substitution and elimination. I am going to use the substitution method.  

Your system of equation is 5x+3y=6 and -3x-3y=6

5x+3y=6

-5x  -5x

3y=6-5x

Now you substittion the first equation to the sencond one. It will look like:

-3x-(6-5x)=6

-3x-6+5x=6

2x-6=6

+6    +6

2x=12

Divide both sides by 2 and you get

x=6

Now lets substiton x inot the orgianl equation.

-3(6)-3y=6

-18-3y=6

+18    +18

-3y=24

Divide both sides by -3

y=-8

So, the answer is x=6, y=-8

Mason download an app for four dollars. The app has a monthly fee of three dollars per month. Tatum download the same app that cost $20 and has monthly fees of one dollar per month how many months before Mason and Tatum had paid the same amount?

Answers

Step-by-step explanation:

step 1. let Mason = m and Tatum = t

step 2. m = 3x + 4 ($3/month and one time $4)

step 3. t = 1x + 20 ($1/month and one time $20)

step 4. 3x + 4 = x + 20 (set equations equal)

step 5. 2x = 16 (collect like terms)

step 6. x= 8 (answer: 8 months)

Find the sum of -7x^2-6x+9−7x 2 −6x+9 and -3x^2-x+7−3x 2 −x+7. show work

Answers

Let's simplify step-by-step.

−7x2−6x+9−7x2−6x+9

=−7x2+−6x+9+−7x2+−6x+9

Combine Like Terms:

=−7x2+−6x+9+−7x2+−6x+9

=(−7x2+−7x2)+(−6x+−6x)+(9+9)

=−14x2+−12x+18

Answer:

=−14x2−12x+18

Let's simplify step-by-step.

−3x2−x+7−3x2−x+7

=−3x2+−x+7+−3x2+−x+7

Combine Like Terms:

=−3x2+−x+7+−3x2+−x+7

=(−3x2+−3x2)+(−x+−x)+(7+7)

=−6x2+−2x+14

Answer:

=−6x2−2x+14

Answer: -10x^2-7x+16

Step-by-step explanation: (-7x^2-6x+9)+(-3x^2-x+7)  

-7x^2-6x+9-3x^2-x+7

Nothing to distribute.

Combine like terms:

Final Answer: -10x^2-7x+16

Tori bought one share of Macy's stock on Nov 1, 2016 for $33.38.
Four years later, she sold it and the closing price for that day was
$5.09. a) How much money did she gain/lose with this stock?
b) What is her rate of return?

Answers

Tori's loss from the stock will be $28.29 and her rate of return will be -84.75%.

Since Tori bought one share of Macy's stock on Nov 1, 2016 for $33.38 and she sold it and the closing price for that day was $5.09, her loss will be;

= $5.09 - $33.38

= -$28.29

Her rate of return will be:

= -28.29/33.38 × 100

= -84.75%

Read related link on:

https://brainly.com/question/20126641

Find the slope and y-intercept of the line

y=-1/5x+9

Answers

Answer:

Step-by-step explanation:

Slope and y-intercept form of a line: y =mx + b

m -> Slope;     b -> y-intercept

y = (-1/5)x + 9

Slope = -1/5

y-intercept = 9

will mark brainliest pls help

Answers

Answer:

y = 90 and z = 62.........

Y=90 z=62 I’m pretty sure that’s correct

Deion finds that the world's population is 7.048 billion and is increasing at a rate of 1.1% per year. Which function
would Deion use to model the growing population in billions, P(t), based on the number of years, t, after this year?

Answers

Answer:

Step-by-step explanation:

I think it’s a

David's money is three times Siti's money. Farid has RM 150 less than Siti. The total amount of their money is RM 2 000. How much do each of them have? ​

Answers

Answer:

Siti's money = RM 430

David = RM 1,290

Farid = RM 280

Step-by-step explanation:

Let

Siti's money = x

David = 3x

Farid = x - 150

Total of their money = RM 2 000

x + 3x + (x - 150) = 2000

4x + x - 150 = 2000

5x = 2000 + 150

5x = 2,150

x = 2,150/5

x = RM 430

Siti's money = x

= RM 430

David = 3x

= 3(430)

= RM 1,290

Farid = x - 150

= 430 - 150

= RM 280

Solve algebraically the simultaneous equations
x^2 – 4y^2 = 5
3x + 4y = 13
Please correctly pair your answers and write them on separate lines.

Answers

Step-by-step explanation:

There are two page above i think it be useful

Thanks for this question :)

please answer will give brainliest

Answers

Answer:

50 im sorry if im wrong

Step-by-step explanation:

Answer:

225

Step-by-step explanation:

15 * 15 = 225

225 / 2 =

Other Questions
I need help with math if you don't know the answer get out of the question and don't answer it and give the wrong answer or some link or even say "I don't know" or anything like that it's not fair to me cause it a waste of my points and time I will report your if you do that :) have a nice day and stay safe. PEOPLE WORK HARD FOR THERE POINTS IF YOU DON'T KNOW How can you check that k 11 is the solution set to the inequality k + 14 25? Can hardware work without software? Why? An outdoor market common in the Middle East with goods, and especially ornamental carpets, is known as a In JKL, J = 7.9 inches, K = 2 inches and L =9.8 inches. Find the measure of K to the nearest degree In the diagram, which angle is an alternate exterior angle with angle 4? How would I solve this? Mr. Fernandez uses 7 boards that are 4 feet long and 6 inches wide to make on bookshelf. If he buys lumber in lengths of 8 feet with a width of 12 inches, how many pieces of lumber does he need to purchase to make 5 bookshelves? For a bronze alloy, the stress at which plastic deformation begins is 274 MPa and the modulus of elasticity is 118 GPa. (a) What is the maximum load that may be applied to a specimen having a cross-sectional area of 316 mm2 without plastic deformation? (b) If the original specimen length is 132 mm, what is the maximum length to which it may be stretched without causing plastic deformation? : 1. , .2. , , . the one-to-one functions g and h are defined as followg={(-2, -6), (3, -3), (6, -9), (9, 6)} h(x) = 2x-3 What is the ultimate purpose of the nuclear reaction in a nuclear power plant In how many ways can youselect 3 books to bring onvacation from a group of 11books? 139 / 263 MarksThe pictogram shows the amounts Judy saves in four months.AugustKey:Septemberrepresents 20OctoberNovembera)Work out the total amount of money that Judy saves in the four months.b)Judy spends 172 on Christmas presents.How much money does she have left? plzz answer ALL questions pllzzzz for 66 points Find the scale factor. What role does technology play in communication? Physical quantities are generally not purely numerical: They have a particular dimension or combination of dimensions associated with them. Thus, your height is not 74, but rather 74 inches, often expressed as 6 feet 2 inches. Although feet and inches are different units they have the same dimension--length.Required:a. Find the dimensions [V] of volume.b. Find the dimensions [v] of speed. can someone answer this one for me? Doughnuts are sold in bag and cartons. A bag holds 4 doughnuts and a carton holds 10 doughnuts. Tome buys b bags of doughnuts and c cartons of doughnuts. He buys a total of t doughnuts. Write down the formula for t in terms of b and c